1. Find the Perimeter AND Area of the figure
below.
2 ft
5 ft
2 ft
5 ft

1. Find The Perimeter AND Area Of The Figurebelow.2 Ft5 Ft2 Ft5 Ft

Answers

Answer 1

Answer:

A = 16 ft^2

P = 20 ft

Step-by-step explanation:

P = perimeter

A = area

STEP 1: divide the shape into rectangles

Rectangle 1: 2ft*3ft

Rectangle 2: 2ft*5ft

STEP 2: Find the area of each rectangle

Equation for area of a rectangle = bh

Rectangle 1: b = 2, h = 3

Rectangle 2: b = 2, h = 5

(2 * 3) + (2 * 5)

6 + 10

16 ft^2

Now, we have to find the perimeter

STEP 1:  Find the unknown side lengths.

To find the lengths of the sides not labeled, you have to use the lengths of the sides we already know.

The length of one parallel side is 5, and the length of another parallel side is 2. The length of the unknown side starts at the same place as the top of the side length that is 5, and ends at the top of the side length that is 2. This means that we have to subtract 2 from 5 in order to find the unknown side length.

STEP 2: Add up all the side lengths

P = 2 + 5 + 5 + 2 + 3 + 3

P = 20 ft

Don't forget to label your answers!!

I hope this made sense, it's is a little hard to explain in simple terms without being able to draw, but I hope it helped.


Related Questions

Complete the information for that object by making estimates using appropriate units of measurement of the dimensions and by getting the actual measurements using an appropriate measuring instrument. ​

Answers

Answer:

hlo how are u?whats ur day is going

The following data was collected to explore how the average number of hours a student studies per night and the student's GPA affect their ACT score. The dependent variable is the ACT score, the first independent variable (x1)is the number of hours spent studying, and the second independent variable (x2) is the student's GPA
Effects on ACT Scores
Study Hours GPA ACT Score
5 4 27
5 2 18
5 3 18
1 3 20
2 4 21
Step 1 of 2: Find the p-value for the regression equation that fits the given data. Round your answer to four decimal places.
Step 2 of 2: Determine if a statistically significant linear relationship exists between the independent and dependent variables at the 0.01 level of significance. If the relationship is statistically significant, identify the multiple regression equation that best fits the data, rounding the answers to three decimal places. Otherwise, indicate that there is not enough evidence to show that the relationship is statistically significant.

Answers

Answer:

Pvalue = 0.1505

y = 0.550x1 + 3.600x2 + 7.300

Step-by-step explanation:

Given the data :

Study Hours GPA ACT Score

5 4 27

5 2 18

5 3 18

1 3 20

2 4 21

Using technology, the Pvalue obtained using the Fratio :

F = MSregression / MSresidual = 30.228571/ 8.190476 = 3.69

The Pvalue for the regression equation is:

Using the Pvalue from Fratio calculator :

F(1, 3), 3.69 = 0.1505

Using the Pvalue approach :

At α = 0.01

Pvalue > α ; Hence, we fail to reject H0 and conclude that ; There is not enough evidence to show that the relationship is statistically significant.

The regression equation :

y = A1x1 + A2x2 +... AnXn

y = 0.550x1 + 3.600x2 + 7.300

x1 and x2 are the predictor variables ;

y = predicted variable

In the diagram, WZ=StartRoot 26 EndRoot.

On a coordinate plane, parallelogram W X Y Z is shown. Point W is at (negative 2, 4), point X is at (2, 4), point Y is at (1, negative 1), and point Z is at (negative 3, negative 1).

What is the perimeter of parallelogram WXYZ?

units
units
units
units

Answers

Answer:

[tex]P = 8 + 2\sqrt{26}[/tex]

Step-by-step explanation:

Given

[tex]W = (-2, 4)[/tex]

[tex]X = (2, 4)[/tex]

[tex]Y = (1, -1)[/tex]

[tex]Z = (-3,-1)[/tex]

Required

The perimeter

First, calculate the distance between each point using:

[tex]d = \sqrt{(x_1 - x_2)^2 + (y_1 -y_2)^2[/tex]

So, we have:

[tex]WX = \sqrt{(-2- 2)^2 + (4-4)^2 } =4[/tex]

[tex]XY = \sqrt{(2- 1)^2 + (4--1)^2 } =\sqrt{26}[/tex]

[tex]YZ = \sqrt{(1- -3)^2 + (-1--1)^2 } =4[/tex]

[tex]ZW = \sqrt{(-3--2)^2 + (-1-4)^2 } =\sqrt{26}[/tex]

So, the perimeter (P) is:

[tex]P = 4 + \sqrt{26} + 4 + \sqrt{26}[/tex]

[tex]P = 8 + 2\sqrt{26}[/tex]

Answer:

its D.

Step-by-step explanation:

took test

Find the value of x.

Answers

Answer:

x = 3

Step-by-step explanation:

A midsegment in a trapezoid is formed when one connects the midpoints of the two legs (non-parallel sides) in a trapezoid. The midsegment theorem states that the length of the midsegment is equal to the average of the two bases (that is the parallel sides).

One can apply the midsegment theorem here by stating the following;

[tex]\frac{(YZ)+(TM)}{2}=PW[/tex]

Substitute,

[tex]\frac{23+11x+2}{2}=29[/tex]

Simplify,

[tex]\frac{25+11x}{2}=29[/tex]

Inverse operations,

[tex]\frac{25+11x}{2}=29[/tex]

[tex]25+11x=58\\\\11x = 33\\\\x = 3[/tex]

A town recently dismissed 5 employees in order to meet their new budget reductions. The town had 4 employees over 50 years of age and 16 under 50. If the dismissed employees were selected at random, what is the probability that no more than 1 employee was over 50

Answers

Answer:

0.7513 = 75.13% probability that no more than 1 employee was over 50

Step-by-step explanation:

The employees are chosen from the sample without replacement, which means that the hypergeometric distribution is used to solve this question.

Hypergeometric distribution:

The probability of x successes is given by the following formula:

[tex]P(X = x) = h(x,N,n,k) = \frac{C_{k,x}*C_{N-k,n-x}}{C_{N,n}}[/tex]

In which:

x is the number of successes.

N is the size of the population.

n is the size of the sample.

k is the total number of desired outcomes.

Combinations formula:

[tex]C_{n,x}[/tex] is the number of different combinations of x objects from a set of n elements, given by the following formula.

[tex]C_{n,x} = \frac{n!}{x!(n-x)!}[/tex]

In this question:

4 + 16 = 20 employees, which means that [tex]N = 20[/tex]

4 over 50, which means that [tex]k = 4[/tex]

5 were dismissed, which means that [tex]n = 5[/tex]

What is the probability that no more than 1 employee was over 50?

Probability of at most one over 50, which is:

[tex]P(X \leq 1) = P(X = 0) + P(X = 1)[/tex]

In which

[tex]P(X = x) = h(x,N,n,k) = \frac{C_{k,x}*C_{N-k,n-x}}{C_{N,n}}[/tex]

[tex]P(X = 0) = h(0,20,5,4) = \frac{C_{4,0}*C_{16,5}}{C_{20,5}} = 0.2817[/tex]

[tex]P(X = 1) = h(1,20,5,4) = \frac{C_{4,1}*C_{16,4}}{C_{20,5}} = 0.4696[/tex]

Then

[tex]P(X \leq 1) = P(X = 0) + P(X = 1) = 0.2817 + 0.4696 = 0.7513[/tex]

0.7513 = 75.13% probability that no more than 1 employee was over 50

I WILL MARK BRAINLIEST PLEASE HELP! This graph represents f(x), and g(x) = -7x + 8.



Which statement about these functions is true?

A.
Function f(x) is increasing, and g(x) is decreasing.
B.
Function f(x) is decreasing, and g(x) is increasing.
C.
Functions f(x) and g(x) are both decreasing.
D.
Functions f(x) and g(x) are both increasing.

Answers

Answer:

A

Step-by-step explanation:

ITS OPTION (A)

PLZ MARK ME BRAINLIEST..

In your office desk drawer you have 10 different flavors of fruit leather. How many distinct flavor groupings can you make with your fruit leather stash?​

Answers

100

You have 10 different flavors. Just multiply the ten with another ten.

find the length of side AB

Answers

Answer:

AB = 5.6 cm

Step-by-step explanation:

Reference angle (θ) = 62°

Hypotenuse = 12 cm

Adjacent = AB

Apply the trigonometric ratio formula, CAH, which is:

Cos θ = Adj/Hyp

Plug in the values

Cos 62° = AB/12

12*Cos 62° = AB

5.63365876 = AB

AB = 5.6 cm (1 decimal place)

In order to win a prize, Heather randomly draws two balls from a basket of 40. There are 25 blue balls, and the rest are green balls. Of the blue balls, 12% are winning balls. Of the green balls, 20% are winning balls. Calculate the expected number of winning balls that Heather draws.

Answers

Answer:

The expected number of winning balls that Heather draws is 0.3.

Step-by-step explanation:

The balls are chosen without replacement, which means that the hypergeometric distribution is used to solve this question.

Hypergeometric distribution:

The probability of x successes is given by the following formula:

[tex]P(X = x) = h(x,N,n,k) = \frac{C_{k,x}*C_{N-k,n-x}}{C_{N,n}}[/tex]

In which:

x is the number of successes.

N is the size of the population.

n is the size of the sample.

k is the total number of desired outcomes.

Combinations formula:

[tex]C_{n,x}[/tex] is the number of different combinations of x objects from a set of n elements, given by the following formula.

[tex]C_{n,x} = \frac{n!}{x!(n-x)!}[/tex]

Expected value of the hypergeometric distribution:

The expected value is given by:

[tex]E(X) = \frac{nk}{N}[/tex]

Expected number of blue and green balls:

40 balls, which means that [tex]N = 40[/tex]

2 are chosen, which means that [tex]n = 2[/tex]

25 are blue, which means that [tex]k = 25[/tex]

So

[tex]E(X) = \frac{nk}{N} = \frac{25(2)}{40} = 1.25[/tex]

1.25 balls are expected to be blue and 2 - 1.25 = 0.75 green.

Of the blue balls, 12% are winning.

Of the green balls, 20% are winning.

Calculate the expected number of winning balls that Heather draws.

[tex]E_w = 1.25*0.12 + 0.75*0.2 = 0.3[/tex]

The expected number of winning balls that Heather draws is 0.3.

The solution of this equation has an error. Which of the following steps has the error? 18 − (3x + 5) = 8
Step 1: 18 − 3x + 5 = 8
Step 2: -3x + 23 = 8
Step 3: -3x = -15
Step 4: x = 5



Step 1 Step 2 Step 3 Step 4. ?

Answers

Answer:

Step 1

Because the number in front of the bracket is 1 and it is also affected by the negative sign(-),5 is supposed to be negative not positive because (negative by positive is negative)

And since the first step has an error in it,the remaining steps would also be wrong.

Suppose f(x,y,z) = x2 + y2 + z2 and W is the solid cylinder with height 7 and base radius 2 that is centered about the z-axis with its base at z = −2. Enter θ as theta.

A) As an iterated integral, ∭WfdV = ∫BA∫DC∫FE dzdrdθ with limits of integration.

B) Evaluate the integral.

Answers

In cylindrical coordinates, W is the set of points

W = {(r, θ, z) : 0 ≤ r ≤ 2 and 0 ≤ θ ≤ 2π and -2 ≤ z ≤ 5}

(A) Then the integral of f(x, y, z) over W is

[tex]\displaystyle\iiint_W(x^2+y^2+z^2)\,\mathrm dV = \int_0^{2\pi}\int_0^2\int_{-2}^5r(r^2+z^2)\,\mathrm dz\,\mathrm dr\,\mathrm d\theta[/tex]

(B)

[tex]\displaystyle \int_0^{2\pi}\int_0^2\int_{-2}^5r(r^2+z^2)\,\mathrm dz\,\mathrm dr\,\mathrm d\theta = 2\pi \int_0^2\int_{-2}^5(r^3+rz^2)\,\mathrm dz\,\mathrm dr \\\\\\= 2\pi \int_0^2\left(zr^3+\frac13rz^3\right)\bigg|_{z=-2}^{z=5}\,\mathrm dr \\\\\\= 2\pi \int_0^2\left(\frac{133}3r+7r^3\right)\,\mathrm dr \\\\\\= 2\pi \left(\frac{133}6r^2+\frac74r^4\right)\bigg|_{r=0}^{r=2} \\\\\\= 2\pi \left(\frac{110}3\right) = \boxed{\frac{220\pi}3}[/tex]

Plz help. I’m finding surface area. I need the answer in units. Thank you.

Answers

Answer:

C. 17 units

Step-by-step explanation:

Surface area of rectangular prism is given as:

A = 2lw + 2lh + 2wh

A = 930 square units

l = 12 units

h = 9 units

w = ? (We're to find the width)

Plug in the value into the formula

930 = 2*12*w + 2*12*9 + 2*w*9

930 = 24w + 216 + 18w

Add like terms

930 - 216 = 42w

714 = 42w

Divide both sides by 42

714/42 = 42w/42

17 = w

w = 17 units

What are the domain and range of the function represented by the set of
ordered pairs?
{(-16, 0), (-8, -11), (0, 12), (12,4)}

Answers

Answer:

domain:-16,-8,0,12

range:0,-11,12,14

PLEASE HELP!!! Which number is a solution of the inequality x less-than negative 4? Use the number line to help answer the question. A number line going from negative 9 to positive 1.

Answers

Answer:

is it going to be 10.5

Step-by-step explanation:

I do not have any explanation

Answer: 0 (zero)

Step-by-step explanation:

Start Learning & start growing! edge2023

*DROPS THE MIC*

A large container holds 4 gallons of chocolate milk that has to be poured into bottles. Each bottle holds 2 pints.
If the ratio of gallons to pints is 1: 8,
bottles are required to hold the 4 gallons of milk.

Answers

Answer:

64 Bottles

Step-by-step explanation:

that is the procedure above

The parametric equations for the paths of two projectiles are given. At what rate is the distance between the two objects changing at the given value of t? (Round your answer to two decimal places.) x1 = 10 cos(2t), y1 = 6 sin(2t) First object x2 = 4 cos(t), y2 = 4 sin(t) Second object t = π/2

Answers

Answer:

-  [tex]\frac{4}{\sqrt{29} }[/tex]

Step-by-step explanation:

The equations for the 1st object :

x₁ = 10 cos(2t),  and  y₁ = 6 sin(2t)

2nd object :

x₂ = 4 cos(t), y₂ = 4 sin(t)

Determine rate at which distance between objects will continue to change

solution Attached below

Distance( D )  = [tex]\sqrt{(10cos2(t) - 4cos(t))^2 + (6sin2(t) -4sin(t))^2}[/tex]

hence; dD/dt = - [tex]\frac{4}{\sqrt{29} }[/tex]

Find the solution of the differential equation that satisfies the given initial condition. (dP)/(dt)

Answers

Answer:

[tex]P = (\frac{1}{3}t^\frac{3}{2} + \sqrt 2 - \frac{1}{3})^2[/tex]

Step-by-step explanation:

Given

[tex]\frac{dP}{dt} = \sqrt{Pt[/tex]

[tex]P(1) = 2[/tex]

Required

The solution

We have:

[tex]\frac{dP}{dt} = \sqrt{Pt[/tex]

[tex]\frac{dP}{dt} = (Pt)^\frac{1}{2}[/tex]

Split

[tex]\frac{dP}{dt} = P^\frac{1}{2} * t^\frac{1}{2}[/tex]

Divide both sides by [tex]P^\frac{1}{2}[/tex]

[tex]\frac{dP}{ P^\frac{1}{2}*dt} = t^\frac{1}{2}[/tex]

Multiply both sides by dt

[tex]\frac{dP}{ P^\frac{1}{2}} = t^\frac{1}{2} \cdot dt[/tex]

Integrate

[tex]\int \frac{dP}{ P^\frac{1}{2}} = \int t^\frac{1}{2} \cdot dt[/tex]

Rewrite as:

[tex]\int dP \cdot P^\frac{-1}{2} = \int t^\frac{1}{2} \cdot dt[/tex]

Integrate the left hand side

[tex]\frac{P^{\frac{-1}{2}+1}}{\frac{-1}{2}+1} = \int t^\frac{1}{2} \cdot dt[/tex]

[tex]\frac{P^{\frac{-1}{2}+1}}{\frac{1}{2}} = \int t^\frac{1}{2} \cdot dt[/tex]

[tex]2P^{\frac{1}{2}} = \int t^\frac{1}{2} \cdot dt[/tex]

Integrate the right hand side

[tex]2P^{\frac{1}{2}} = \frac{t^{\frac{1}{2} +1 }}{\frac{1}{2} +1 } + c[/tex]

[tex]2P^{\frac{1}{2}} = \frac{t^{\frac{3}{2}}}{\frac{3}{2} } + c[/tex]

[tex]2P^{\frac{1}{2}} = \frac{2}{3}t^\frac{3}{2} + c[/tex] ---- (1)

To solve for c, we first make c the subject

[tex]c = 2P^{\frac{1}{2}} - \frac{2}{3}t^\frac{3}{2}[/tex]

[tex]P(1) = 2[/tex] means

[tex]t = 1; P =2[/tex]

So:

[tex]c = 2*2^{\frac{1}{2}} - \frac{2}{3}*1^\frac{3}{2}[/tex]

[tex]c = 2*2^{\frac{1}{2}} - \frac{2}{3}*1[/tex]

[tex]c = 2\sqrt 2 - \frac{2}{3}[/tex]

So, we have:

[tex]2P^{\frac{1}{2}} = \frac{2}{3}t^\frac{3}{2} + c[/tex]

[tex]2P^{\frac{1}{2}} = \frac{2}{3}t^\frac{3}{2} + 2\sqrt 2 - \frac{2}{3}[/tex]

Divide through by 2

[tex]P^{\frac{1}{2}} = \frac{1}{3}t^\frac{3}{2} + \sqrt 2 - \frac{1}{3}[/tex]

Square both sides

[tex]P = (\frac{1}{3}t^\frac{3}{2} + \sqrt 2 - \frac{1}{3})^2[/tex]

Please help me >_< will give out brainliest

Answers

Answer:  1080

====================================================

Explanation:

We have an octagon because there are n = 8 sides. The diagram below shows one way to number the sides so you can count them efficiently (without missing any or double counting any).

----------------

Plug n = 8 into the formula below

S = 180(n-2)

S = 180(8-2)

S = 180(6)

S = 1080

The 8 interior angles add up to 1080 degrees.

Please help me in this question

Answers

Answer:

3/8

Step-by-step explanation:

the total number of possible results is 4×4=16.

out of these 16 only the results

1 2

1 3

1 4

2 2

2 3

3 2

are desired results. these are 6.

so the probability of a desired result is 6/16 = 3/8

If 128x is a perfect square number what is the least value of x

Please answer the question fast​

Answers

Answer:

in a square all sides are equal so x has to equal

128

Hope This Helps!!!

For this problem what I did was add all the measurements and I got 48 m. However, it is wrong. How do I go about solving the perimeter then?

Answers

9514 1404 393

Answer:

  66 m

Step-by-step explanation:

The perimeter is the sum of the measures of all of the sides. There are two side measures that are missing from the diagram.

The missing horizontal measure is ...

  17 m - 8 m = 9 m

The missing vertical measure is ...

  16m -7 m = 9 m.

If you add these to the sum you already calculated, you will get the correct answer:

  48 m + 9 m + 9 m = 66 m . . . perimeter of the figure

_____

If you're paying attention, you see that the sum of the measures of the two shorter horizontal segments is the same as the measure of the longer horizontal segment. Likewise, the sum of the measurements of the two shorter vertical segments is the same as that of the longer vertical segment.

In other words, the perimeter of this (and any) L-shaped figure is the same as the perimeter of a rectangle having the same horizontal and vertical dimensions as the long sides of the figure.

  P = 2(17 m +16 m) = 2(33 m) = 66 m

Exactly how many planes contain points J, K, and N?
a - 0
b - 1
c - 2
d - 3

Answers

A- 0 ..... one plane contains k and n but there isn’t a plane that contains all three points

Jill has 32 crayons. She loses 4 of the crayons. How many are left?

Answers

Answer:

the answer here is d

the answer is d

Answer:

28

Step-by-step explanation:

Total number of crayons = 32

Number of crayons lost = 4

Therefore, number of crayons she is left with is : 32 - 4 = 28

Working :  

    [tex]32\\04 - \\\overline{28}[/tex]

The asymptote of the function f(x) = 3x + 1 – 2 is . Its y-intercept is

Answers

Answer:

-1

Step-by-step explanation:

1-2=-1

y=mx+b

b= y intercept

Answer:

-1

Step-by-step explanation:

Find a power series representation for the function. (Assume a>0. Give your power series representation centered at x=0 .)
f(x)=x2a7−x7

Answers

Answer:

Step-by-step explanation:

Given that:

[tex]f_x = \dfrac{x^2}{a^7-x^7}[/tex]

[tex]= \dfrac{x^2}{a^7(1-\dfrac{x^7}{a^7})}[/tex]

[tex]= \dfrac{x^2}{a^7}\Big(1-\dfrac{x^7}{a^7} \Big)^{-1}[/tex]

since  [tex]\Big((1-x)^{-1}= 1+x+x^2+x^3+...=\sum \limits ^{\infty}_{n=0}x^n\Big)[/tex]

Then, it implies that:

[tex]\implies \dfrac{x^2}{a^7} \sum \limits ^{\infty}_{n=0} \Big(\Big(\dfrac{x}{a} \Big)^{^7} \Big)^n[/tex]

[tex]= \dfrac{x^2}{a^7} \sum \limits ^{\infty}_{n=0} \Big(\dfrac{x}{a} \Big)^{^{7n}}[/tex]

[tex]= \dfrac{x^2}{a^7} \sum \limits ^{\infty}_{n=0} \Big(\dfrac{x^{7n}}{a^{7n}} \Big)}[/tex]

[tex]\mathbf{= \sum \limits ^{\infty}_{n=0} \dfrac{x^{7n+2}}{a^{7n+7}} }}[/tex]

Roulette is a casino game that involves spinning a ball on a wheel that is marked with numbered squares that are red, black, or green. Half of the numbers 1 - 36 are colored red and half are black and the numbers 0 and 00 are green. Each number occurs only once on the wheel. What is the probability of landing on an even number and a number greater than 17? (A number is even if it is divisible by 2. 0 and 00 are considered even as well.)​

Answers

Answer:

the wording (punctuation) of the question can lead to different interpretations....

I assume that the question was >17 & even which is "5/19",

BUT... it can also be read as two questions

first >17 which is "10/19"

and second an even number which is "9/19"

BUT !!! I think that the question answer is 5/19

Step-by-step explanation:

Even Number  = 18/38 = 9/19

greater 17 = 20/38 = 10/19

Even & greater 17 = 10/38 = 5/19

A school contains 140 boys and 160 girls. what is the ratio of boys to girls?
I need full working out please

Answers

Answer:

7 : 8

Step-by-step explanation:

that is the procedure above

SCALCET8 3.9.015. A street light is mounted at the top of a 15-ft-tall pole. A man 6 ft tall walks away from the pole with a speed of 4 ft/s along a straight path. How fast is the tip of his shadow moving when he is 35 ft from the pole

Answers

Answer:

[tex]X=6.67ft/s[/tex]

Step-by-step explanation:

From the question we are told that:

Height of pole [tex]H_p=15[/tex]

Height  of man [tex]h_m=6ft[/tex]

Speed of Man [tex]\triangle a =4ft/s[/tex]

Distance from pole [tex]d=35ft[/tex]

Let

Distance from pole to man=a

Distance from man to shadow =b

Therefore

 [tex]\frac{a+b}{15}=\frac{b}{6}[/tex]

 [tex]6a+6b=15y[/tex]

 [tex]2a=3b[/tex]

Generally the equation for change in velocity is mathematically given by

 [tex]2(\triangle a)=3(\triangle b )[/tex]

 [tex]2*4=3(\triangle b)[/tex]

 [tex]\triangle a=\frac{8}{3}[/tex]

Since

The speed of the shadow is given as

 [tex]X=\triangle b+\triangle a[/tex]

 [tex]X=4+8/3[/tex]

 [tex]X=6.67ft/s[/tex]

Find the perimeter of a football field which measures 90m by 60m

Answers

Hello!

[tex]\large\boxed{P = 300m}[/tex]

Use the following formula for the perimeter:

P = 2l + 2w, where:

l = length

w = width

Therefore:

P = 2(90) + 2(60)

Simplify:

P = 180 + 120 = 300 m

Answer:

well how about you use common sense 100 yards long on each side 200 yards then add 5o yards since the the that is how wide it is then add another 50 and you get 300 yards then convert that to meters

PLSHELPASAPDFFFFFFFFFFFFFFFFFFFFFFFFFF

Answers

im struggling with the same one

Other Questions
How did Frank Lloyd Wright believe a home should interact with the landscape? a. the house should be in the landscape b. the house should overshadow the landscape c. the landscape should overshadow the house d. none of the above PLS HELP I WILL GIVE BRAINLIESTKimberly will be riding her bike to school this year. The distance from her house to the end of the street is 1/6 mile. The distance from the end of the street to school is 3/8 mile. About how far is Kimberly's house from school? I have a lot of algebra problems. Someone help me even with this one please! can anyone help with this please !!!! Which of the following spheres of the Earth is most directly affected by earthquakes and volcanoes? O A. Atmosphere O B. Lithosphere O C. Biosphere O D. Hydrosphere . A small home business is set up with an investment of Birr 1,000,000 for equipment. The business manufactures a product at a cost of Birr 60 per unit. If the product sells for Birr 140, how many units must be sold before the business breaks even? The equation of line r is y = 1/2 * x + 1 line runs parallel to line r and passes through (2, 5) what would be the equation of line 8 ?help please If 3 3/4m of cloth was used for one suit, how many suits can be made with 30m cloth Esther's mother earned $8 more than 3 times the amount Esther earned during the summer. If Esther'smother earned m dollars, which of the following represents the amount, in dollars, that Esther earned duringthe summer? Work out the area of this circle.Give your answer in terms ofand state its units.units:Submit ANSWEI6 mmPlss help due in very soon The top and bottom ends of a windshield wiper blade are R = 24 in. and r = 14 in., respectively, from the pivot point. While in operation, the wiper sweeps through 135. Find the area swept by the blade. (Round your answer to the nearest whole number.) Use a half angle identity to find the exact value of tan 5pi/12a. 2+squared3/2b. 2-squared3/2C.2+squared 3D.2-squared3. Please select the best answer from the choices provided Date Page The male population of a village is 9840 and the female population is 8965. Find the total population of the village ii) How many more males are there than females The specific actions to be taken in implementing a strategy. need help with algebra problem Find the measure of the missing angle using the exterior angle sum theorm que contiene el condn? HURRY NEED ASAP TRYNA FINISH SUMMER SCHOOL LOL, I WILL MARK BRAINLIEST :)) PICTURE IS THERE FOR U If the signal is going through a 2 MHz Bandwidth Channel, what will be the maximum bit rate that can be achieved in this channel? What will be the appropriate signal level? can anyone help me here asapp,, I am in this question for nearly an hour